\lecture{19}{2023-06-22}{} \subsection{Uniform Integrability} \begin{example} Let $\Omega = [0,1]$, $\cF = \cB$ and $\bP = \lambda \defon{[0,1]}$. Consider $X_n \coloneqq n \One_{(0,\frac{1}{n}))}$. We know that $X_n \xrightarrow{n \to \infty} 0$ a.s., however $\bE[X_n] = \bE[|X_n|] = 1$, hence $X_n$ does not converge in $L^1(\bP)$. Let $\mu_n(\cdot ) = \bP[X_n \in \cdot ]$. Intuitively, for a series that converges in probability, for $L^1$-convergence to hold we somehow need to make sure that probability measures don't assign mass far away from $0$. This will be made precise in the notion of uniform integrability. \end{example} \begin{goal} We want to show that uniform integrability and convergence in probability is equivalent to convergence in $L^1$. \end{goal} \begin{definition} \label{def:ui} A sequence of random variables $(X_n)_n$ is called \vocab{uniformly integrable} (UI), if \[\forall \epsilon > 0 .~\exists K > 0 .~ \forall n.~ \bE[|X_n| \One_{\{|X_n| > K\} }] < \epsilon.\] Similarly, we define uniformly integrable for sets of random variables. \end{definition} \begin{example} $X_n \coloneqq n \One_{(0,\frac{1}{n})}$ is not uniformly integrable. \end{example} There is no nice description of uniform integrability. However, some subsets can be easily described, e.g. \begin{fact}\label{lec19f1} If $(X_n)_{n \ge 1}$ is a sequence bounded in $L^{1 + \delta}(\bP)$ for some $\delta > 0$ (i.e.~$\sup_n \bE[|X_n|^{1+\delta}] < \infty$), then $(X_n)_n$ is uniformly integrable. \end{fact} \begin{proof} Let $\epsilon > 0$. Let $p \coloneqq 1 + \delta > 1$. Choose $q$ such that $\frac{1}{p} + \frac{1}{q} = 1$. Then \begin{IEEEeqnarray*}{rCl} \bE[|X_n| \One_{|X_n| > K}] &\le& \bE[|X_n|^p]^{\frac{1}{p}} \bP[|X_n| > k]^{\frac{1}{q}},\\ \end{IEEEeqnarray*} i.e. \begin{IEEEeqnarray*}{rCl} \sup_n\bE[|X_n| \One_{|X_n| > k}] &\le& \underbrace{\sup_n\bE[|X_n|^p]^{\frac{1}{p}}}_{< \infty} \sup_n \underbrace{\bP[|X_n| > K]^{\frac{1}{q}}}_% {\le K^{-\frac{1}{q}} \bE[|X_n|]^{\frac{1}{q}}}\\ \end{IEEEeqnarray*} where we have applied \yaref{thm:markov}. Since $\sup_n \bE[|X_n|^{1+\delta}] < \infty$, we have that $\sup_n \bE[|X_n|] < \infty$ by \yaref{jensen}. Hence for $K$ large enough the relevant term is less than $\epsilon$. \end{proof} \begin{fact}\label{lec19f2} If $(X_n)_n$ is uniformly integrable, then $(X_n)_n$ is bounded in $L^1$. \end{fact} \begin{proof} Take some $\epsilon > 0$ and $K$ such that $\sup_n\bE[|X_n| \One_{|X_n| > K}] < \epsilon$. Then $\sup_n\|X_n\|_{L^1} \le K + \epsilon$. \end{proof} \begin{fact}\label{lec19f3} Suppose $Y \in L^1(\bP)$ and $\sup_n |X_n(\cdot )| \le Y(\cdot )$. Then $(X_n)_n$ is uniformly integrable. \end{fact} \begin{fact}\label{lec19f4} Let $X \in L^1(\bP)$. \begin{enumerate}[(a)] \item $\forall \epsilon > 0 .~ \exists \delta > 0 .~\forall F \in \cF .~ \bP(F) < \delta \implies\int_F |X| \dif \bP < \epsilon$. \item $\forall \epsilon > 0 .~ \exists k \in (0,\infty) .~ \int_{|X| > k} | X| \dif \bP < \epsilon$. \end{enumerate} \end{fact} \begin{proof} \begin{enumerate}[(a)] \item Suppose not. Then for $\delta = 1, \frac{1}{2}, \frac{1}{2^2}, \ldots$ there exists $F_n$ such that $\bP(F_n) <\frac{1}{2^n}$ but $\int_{F_n} |X| \dif \bP \ge \epsilon$. Since $\sum_{n} \bP(F_n) < \infty$, by \yaref{thm:borelcantelli}, \[\bP[\underbrace{\limsup_n F_n}_{\text{\reflectbox{$\coloneqq$}}F}] = 0.\] We have \begin{IEEEeqnarray*}{rCl} \int_F | X| \dif \bP &=& \int |X| \One_F \dif \bP\\ &=& \int \limsup_n (|X| \One_{F_n}) \dif \bP\\ &\overset{\text{Reverse Fatou}}{\ge }& \limsup_n \int |X| \One_{F_n} \dif \bP\\ &\ge & \epsilon \end{IEEEeqnarray*} where the assumption that $X$ is in $L^1$ was used to apply the reverse of Fatou's lemma. % TODO reverse fatou This yields a contradiction since $\bP(F) = 0$. \item We want to apply part (a) to $F = \{ |X| > k\}$. By \yaref{thm:markov}, $\bP(F) \le \frac{1}{k} \bE[|X|]$. Since $\bE[|X|] < \infty$, we can choose $k$ large enough to get $\bP(F) \le \delta$. \end{enumerate} \end{proof} \begin{refproof}{lec19f3} Fix $\epsilon > 0$. We have \[ \bE[|X_n| \One_{|X_n| > k}] \le \bE[|Y| \One_{|Y| > k}] < \epsilon \] for $k$ large enough by \yaref{lec19f4} (b). \end{refproof} \begin{fact}\label{lec19f5} Let $X \in L^1(\bP)$. Then $\bF \coloneqq \{ \bE[X | \cG] : \cG \subseteq \cF \text{ sub-$\sigma$-algebra}\}$ is uniformly integrable. \end{fact} \begin{proof} Fix $\epsilon > 0$. Choose $\delta > 0$ such that \begin{equation} \forall F \in \cF.~ \bP(F) < \delta \implies \bE[|X| \One_F] <\epsilon. \label{lec19eqstar} \end{equation} Let $Y = \bE[X | \cG]$ for some sub-$\sigma$-algebra $\cG$. Then, by \yaref{cjensen}, $|Y| \le \bE[ |X| | \cG]$. Hence $\bE[|Y|] \le \bE[|X|]$. By \yaref{thm:markov}, it follows that $\bP[|Y| > k] < \delta$ for $k > \frac{\bE[|X|]}{\delta}$. Note that $\{|Y| > k\} \in \cG$. We have \begin{IEEEeqnarray*}{rCl} \bE[|Y| \One_{\{|Y| > k\} }] &<& \epsilon \end{IEEEeqnarray*} by \eqref{lec19eqstar}, since $\bP[|Y| > k] < \delta$. \end{proof} \begin{theorem} \label{thm:l1iffuip} Assume that $X_n \in L^1$ for all $n$ and $X \in L^1$. Then the following are equivalent: \begin{enumerate}[(1)] \item $X_n \to X$ in $L^1$. \item $(X_n)_n$ is uniformly integrable and $X_n \to X$ in probability. \end{enumerate} \end{theorem} \begin{proof} (2) $\implies$ (1) Define \begin{IEEEeqnarray*}{rCl} \phi(x) &\coloneqq & \begin{cases} -k, & x \le -k\\ x, & x \in (-k,k)\\ k, & x \ge k. \end{cases} \end{IEEEeqnarray*} $\phi$ is $1$-Lipschitz. % TODO We have \begin{IEEEeqnarray*}{rCl} \int |X_n - X| \dif \bP &\le & \int |X_n - \phi(X_n)| \dif \bP + \int |\phi(X) - X| \dif \bP + \int |\phi(X_n) - \phi(X)| \dif \bP\\ \end{IEEEeqnarray*} We have $\int_{|X_n| > k} \underbrace{|X_n - \phi(X_n)|}_{\le |X_n| + | \phi(X_n)| \le 2 |X_n|} \dif \bP\le \epsilon$ by uniform integrability and \yaref{lec19f4} part (b). Similarly $\int_{|X| > k} |X - \phi(X)| \dif \bP < \epsilon$. Since $\phi$ is Lipschitz, $ X_n \xrightarrow{\bP} X \implies \phi(X_n) \xrightarrow{\bP} \phi(X)$. By the \yaref{thm:boundedconvergence} $|\phi(X_n)| \le k \implies \int | \phi(X_n) - \phi(X)| \dif \bP \to 0$. (1) $\implies$ (2) $X_n \xrightarrow{L^1} X \implies X_n \xrightarrow{\bP} X$ by \yaref{thm:markov} (see \yaref{claim:convimpll1p}). Fix $\epsilon > 0$. We have \begin{IEEEeqnarray*}{rCl} \bE[|X_n|] &=& \bE[|X_n - X + X|]\\ &\le & \epsilon + \bE[|X|]\\ &<& \delta k \end{IEEEeqnarray*} for all $\delta > 0$ and suitable $k$. Hence $\bP[|X_n| > k] < \delta$ by \yaref{thm:markov}. Then by \yaref{lec19f4} part (a) it follows that \[ \int_{|X_n| > k} |X_n| \dif \bP \le \underbrace{\int |X - X_n| \dif \bP}_{< \epsilon} + \int_{|X_n| > k} |X| \dif \bP \le 2 \epsilon. \] \end{proof} \subsection{Martingale Convergence Theorems in \texorpdfstring{$L^p, p \ge 1$}{$Lp, p >= 1$}} Let $(\Omega, \cF, \bP)$ as always and let $(\cF_n)_n$ always be a filtration. \begin{fact}\label{lec19f6} Suppose that $X \in L^p$ for some $p \ge 1$. Then $(\bE[X | \cF_n])_n$ is an $\cF_n$-martingale. \end{fact} \begin{proof} It is clear that $(\bE[X | \cF_n])_n$ is adapted to $(\cF_n)_n$. Let $X_n \coloneqq \bE[X | \cF_n]$. Consider \begin{IEEEeqnarray*}{rCl} \bE[X_n - X_{n-1} | \cF_{n-1}] &=& \bE[\bE[X | \cF_n] - \bE[X | \cF_{n-1}] | \cF_{n-1}]\\ &=& \bE[X | \cF_{n-1}] - \bE[X | \cF_{n-1}]\\ &=& 0. \end{IEEEeqnarray*} \end{proof} \begin{theorem} \label{ceismartingale} Let $X \in L^p$ for some $p \ge 1$ and $\bigcup_n \cF_n \to \cF \supseteq \sigma(X)$. Then $X_n \coloneqq \bE[X | \cF_n]$ defines a martingale which converges to $X$ in $L^p$. \end{theorem} % \todo{Proof ?} \begin{theorem} \label{martingaleisce} Let $p > 1$. Let $(X_n)_n$ be a martingale bounded in $L^p$. Then there exists a random variable $X \in L^p$, such that $X_n = \bE[X | \cF_n]$ for all $n$. In particular, $X_n \xrightarrow{L^p} X$. \end{theorem}